Difference between revisions of "2022 AMC 12A Problems/Problem 15"
Phuang1024 (talk | contribs) (Created page with "==Problem== The roots of the polynomial <math>10x^3 - 39x^2 + 29x - 6</math> are the height, length, and width of a rectangular box (right rectangular prism. A new rectangu...") |
MRENTHUSIASM (talk | contribs) (Redirected page to 2022 AMC 10A Problems/Problem 16) (Tag: New redirect) |
||
(9 intermediate revisions by 6 users not shown) | |||
Line 1: | Line 1: | ||
− | + | #redirect [[2022 AMC 10A Problems/Problem 16]] | |
− | |||
− | |||
− | |||
− | |||
− | |||
− | |||
− | |||
− | |||
− | |||
− | |||
− | |||
− | |||
− | |||
− | |||
− | |||
− | |||
− | |||
− | |||
− | |||
− | |||
− | |||
− | |||
− | |||
− |
Latest revision as of 00:36, 12 November 2022
Redirect to: